You are on page 1of 8

Haleh Zarrini S RRCC MAT121802 202120

Assignment Final Review 1 of 3 due 12/04/2020 at 11:59pm MST

1. (1 point) 2.1/CCD_CCCS_Openstax_AlgTrig_AT-1-001-AS_2_1_23.pg 4. (1 point) 2.5/CCD_CCCS_Openstax_AlgTrig_AT-1-001-AS_2_5_7.pg


Find the coordinates of the midpoint of the line segment that Solve the quadratic equation x2 − 2x − 3 = 0 by factoring. If
joins the two points (13, 3) and (−7, 5). there is more than one correct answer, enter a comma separated
list.
Enter a point as (a, b), including the parentheses.
x= help (numbers)
midpoint: help (points)
Answer(s) submitted:
Answer(s) submitted:


(incorrect)
(incorrect)
Correct Answers:
Correct Answers:
• (3,4) • -1, 3

2. (1 point) 6.6/CCD_CCCS_Openstax_AlgTrig_AT-1-001-AS_6_6_15.pg
5. (1 point) 2.6/RRCC_CCCS_Openstax_AlgTrig_AT-1-001-AS_2_6_28.pg
Give all solutions to the equation, or state that the equation has
Solve the radical equation. Be sure to check for extraneous so-
no solutions by entering “NONE”.
lutions.
If there is more than one solution, enter the answers as a comma-
If you find more than one solution, give your answer as a
separated list.
comma-separated list.
−3 · 6 p+9 − 12 = −30 √ √
3x + 7 + x + 2 = 1.
help (numbers)
x= help (numbers)
Answer(s) submitted:
Answer(s) submitted:


(incorrect)
Correct Answers: (incorrect)
Correct Answers:
• -8
• -2
3. (1 point) 2.1/RRCC_CCCS_Circles_1.pg
Find the center and radius of the following circle. Enter the cen-
ter as an ordered pair. 6. (1 point) 2.7/CCD_CCCS_Openstax_AlgTrig_AT-1-001-AS_2_7_6.pg
(x − 5)2 + (y − 9)2 = 4 Solve the following inequality for x. Enter your answer in inter-
val notation.
center: help (points)
9x − 10 ≤ 3
radius: help (numbers)
help (intervals)
Answer(s) submitted:
Answer(s) submitted:

• •
(incorrect) (incorrect)
Correct Answers: Correct Answers:
• (5,9) • (-infinity,13/9]
• 2

1
7. (1 point) 5.5/RRCC_CCCS_Openstax_AlgTrig_AT-1-001-AS_5_5_16.pg 11. (1 point) 3.4/RRCC_CCCS_Openstax_AlgTrig_AT-1-001-AS_3_4_27.
Use the Factor Theorem to find all the zeros of f (x) = pg
x3 − 13x + 12 given that (x + 4) is a factor. Enter all the zeros as Let h(x) = f (g(x)) = (x − 3)3 . Find f (x) given g(x) = x − 3.
a comma separated list.
f (x) = help (formulas)
Zeros: help (numbers)
Answer(s) submitted:
Answer(s) submitted: •
• (incorrect)
(incorrect) Correct Answers:
Correct Answers: • xˆ3
• 1, 3, -4 12. (1 point) 3.5/RRCC_CCCS_Openstax_AlgTrig_AT-1-001-AS_3_5_27.
8. (1 point) 3.1/RRCC_CCCS_Openstax_AlgTrig_AT-1-001-AS_3_1_37.pg pg
Let f (x) = x2 − 4x. Consider the equation y = (x + 1)2 + 3. By hand without using
technology, sketch a graph of this equation on paper.
Evaluate f (−5) = help (numbers)
Which graph A-D below most closely matches the graph you
Solve f (x) = −4. drew? [?/A/B/C/D]
x= help (numbers)

Answer(s) submitted:


(incorrect)
Correct Answers:
• 45
• 2
9. (1 point) 5.5/RRCC_CCCS_Openstax_AlgTrig_AT-1-001-AS_5_5_8.pg
Use the Remainder Theorem to find f (−5) for f (x) =
5x4 + 24x3 − 8x2 − 10x + 29. A B

f (−5) = help (numbers)

Answer(s) submitted:

(incorrect)
Correct Answers:
• 4
10. (1 point) 3.2/RRCC_CCCS_Openstax_AlgTrig_AT-1-001-AS_3_2_10.
pg
√ C D
Find the domain of f (x) = 4 − 10x. Enter your solution in
interval notation.
(Click on a graph to enlarge it.)
help (intervals)

Answer(s) submitted:
• Consider the equation y = −(x + 2)2 − 1. By hand without
(incorrect) using technology, sketch a graph of this equation on paper.
Correct Answers:
• (-infinity,0.4] Which graph A-D below most closely matches the graph you
drew? [?/A/B/C/D]
2
Answer(s) submitted:

(incorrect)
Correct Answers:
• x*zˆ3/[yˆ(1/4)]

15. (1 point) 3.7/RRCC_CCCS_Openstax_AlgTrig_AT-1-001-AS_3_7_25.


pg
Let the graph of f (x) be given below. Find the following.

A B

C D

(Click on a graph to enlarge it.)


Answer(s) submitted:


(incorrect) (Click on graph to enlarge)
Correct Answers:
• A f (0) = help (numbers)
• D
f −1 (0) = help (numbers)
13. (1 point) 3.5/RRCC_CCCS_Openstax_AlgTrig_AT-1-001-AS_3_5_31.
pg
Answer(s) submitted:
Suppose (11, 3) is a point on the graph of y = f (x). What is a
point that will be on the graph of y = −2 f (x) − 2? •

help (points) (incorrect)
Enter a point as (a, b), including the parentheses. Correct Answers:
Answer(s) submitted: • -2
• • 2
(incorrect)
Correct Answers: 16. (1 point) 5.1/RRCC_CCCS_Openstax_AlgTrig_AT-1-001-AS_5_1_16.
pg
• (11,-8)

14. (1 point) 6.5/CCD_CCCS_Openstax_AlgTrig_AT-1-001-AS_6_5_23.pg


Condense the left-hand side into a single logarithm. Then solve Determine whether f (x) = −2x2 + 8x + 10 has a minimum or
the resulting equation for A. maximum value. Find the value of the minimum or maximum.
Find the axis of symmetry.
log(x) − 14 log(y) + 3 log(z) = log(A)
f (x) has a :
A= help (formulas)
3
• minimum Answer(s) submitted:

• maximum (incorrect)
Correct Answers:
• -5*x-15
The value of the minimum or maximum is: help
(numbers) 19. (1 point) 5.2/RRCC_CCCS_Openstax_AlgTrig_AT-1-001-AS_5_2_53.
pg

The axis of symmetry for f (x) is given by x = For f (x) = −4x3 (−2x − 5), Determine the x-intercepts and the
help (numbers) end behavior. Enter intercepts as a comma separated list of
points. The Desmos graphing tool below may help investigate
the function.
Answer(s) submitted:
x-Intercepts : help (points)


• As x → −∞, f → help (numbers)
(incorrect)
Correct Answers:
As x → ∞, f → help (numbers)
• maximum
• 18
• 2
17. (1 point) 3.2/RRCC_CCCS_Openstax_AlgTrig_AT-1-001-AS_3_2_49. . ¡div id=”calculator” style=”width: 750px; height:
pg
400px;”¿¡/div¿
Given the piecewise function, find the function values.
Answer(s) submitted:
( •
9x − 4 if x < −1 •
f (x) = .
6 − 7x if x ≥ −1 •
(incorrect)
f (−1) = help (numbers) Correct Answers:
• (0,0), (-2.5,0)
f (0) = help (numbers) • infinity
• infinity
f (1) = help (numbers)
20. (1 point) 6.3/RRCC_CCCS_Openstax_AlgTrig_AT-1-001-AS_6_3_7.pg
f (3) = help (numbers) Convert logy (2) = x to exponential form.

Answer(s) submitted: • A. xy = 2

• • B. 2y = x

• • C. y2 = x
(incorrect)
Correct Answers: • D. yx = 2
• 13
• 6 • E. x2 = y
• -1
• -15 • F. None of the above
18. (1 point) 2.2/RRCC_CCCS_Openstax_AlgTrig_AT-1-001-AS_2_2_29.
pg
Find an equation of the line perpendicular to 5y = x − 3 that Convert loga (b) = c to exponential form.
passes through the point (−3, 0).
• A. ba = c
y= help (formulas)
4
• B. cb = a Answer(s) submitted:

• C. ac = b •
(incorrect)
• D. bc = a Correct Answers:
• -x+5
• E. ab = c • -14
24. (1 point) 5.6/RRCC_CCCS_Openstax_AlgTrig_AT-1-001-AS_5_6_9.pg
• F. None of the above
x2 + 4x + 4
Answer(s) submitted: Find the domain of f (x) = 2 . Enter your solution in
x +x−2
• interval notation.

(incorrect) help (intervals)
Correct Answers:
• D Answer(s) submitted:
• C •
(incorrect)
21. (1 point) 5.3/RRCC_CCCS_Openstax_AlgTrig_AT-1-001-AS_5_3_23.
Correct Answers:
pg
• (-infinity,-2) U (-2,1) U (1,infinity)
Find the x-intercept(s) of f (x) = x5 − 7x3 + 12x. Enter inter-
cept(s) as points. If there is more than one point, enter them as 25. (1 point) 5.1/RRCC_CCCS_Openstax_AlgTrig_AT-1-001-AS_5_1_67.
a comma separated list. pg
Find the dimensions of a rectangular corral split into 2 pens
x-intercept(s): help (points) of the same size producing the greatest possible enclosed area
given 100 feet of fencing. Enter the dimensions as a comma
Answer(s) submitted: separated list.

(incorrect) help (numbers)
Correct Answers:
Answer(s) submitted:
• (0,0), (2,0), (-2,0), (1.73205,0), (-1.73205,0)

22. (1 point) 6.5/CCD_CCCS_Openstax_AlgTrig_AT-1-001-AS_6_5_15.p (incorrect)
g Correct Answers:
Use the properties of logarithms to expand the following log- • 16.6667, 25
arithm as much as possible. Rewrite as a sum, difference, or
product of logs. 26. (1 point) 5.6/RRCC_CCCS_Openstax_AlgTrig_AT-1-001-AS_5_6_45.
pg

x5 y6
 −2x2 − 16x − 32
log Let f (x) = . Find the domain, vertical asymp-
z5 4x2 − 4x − 48
totes, horizontal asymptote, and select the correct graph.

Domain: help (intervals)


Answer(s) submitted:
• Vertical Asymptotes: x = help (numbers) (Enter
(incorrect) values as a comma separated list)
Correct Answers:
• 5*log(x)+6*log(y)-5*log(z)
Horizontal Asymptote: y = help (numbers)

23. (1 point) 5.4/RRCC_CCCS_Openstax_AlgTrig_AT-1-001-AS_5_4_7.pg


Use long division to perform the division (−x2 + 2x + 1)/(x +
3). Find the quotient and remainder. Which of the following is the graph of f (x)? [?/A/B/C/D]
(Note, window is from [-10,10] so some of graph may be off the
Quotient: help (formulas) window)

Remainder: help (formulas)


5
28. (1 point) 5.3/RRCC_CCCS_Openstax_AlgTrig_AT-1-001-AS_5_3_32.
pg
Find the zeros and give the multiplicity of each for f (x) =
x2 (x − 3)2 (x + 6)4 .

Note: to be counted as correct, you must get all answers correct.

One zero is x = and has a multiplicity of help (num-


bers)
A B
Another zero is x = and has a multiplicity of: help
(numbers)

The last zero is x = and has a multiplicity of: help


(numbers)

Answer(s) submitted:



C D •


(Click on a graph to enlarge it.) (incorrect)
Answer(s) submitted: Correct Answers:
• • 3
• • 2
• • 0
• • 2
• -6
(incorrect) • 4
Correct Answers:
• (-infinity,-3) U (-3,4) U (4,infinity)
29. (1 point) 7.6/CCD_CCCS_Openstax_IntAlg_IA-2017-000-LC_7_6_33
• 4, -3 9.pg
• -0.5 Solve the rational inequality. Write your answer in interval no-
• C tation.
x−5
27. (1 point) 7.4/CCD_CCCS_Openstax_IntAlg_IA-2017-000-LC_7_4_20 ≥0
3.pg x+5
Solve the rational equation. If there is more than one correct an- help (intervals)
swer, enter a comma separated list. If there is no answer, enter
“NONE”. Answer(s) submitted:
5 6 •
1+ =− 2
p p (incorrect)
Correct Answers:
p= help (numbers) • (-infinity,-5) U [5,infinity)

Answer(s) submitted: 30. (1 point) 2.5/CCD_CCCS_Openstax_AlgTrig_AT-1-001-AS_2_5_25.pg


Solve x2 − 6x − 5 = 0 by completing the square. If there is more

than one correct answer, enter a comma separated list.
(incorrect)
Correct Answers: x= help (numbers)
• -3, -2
Answer(s) submitted:
6
• • (-infinity,infinity)
(incorrect) • (-infinity,-4)
Correct Answers:
• 6.74166, -0.741657
33. (1 point) 3.3/RRCC_CCCS_Openstax_AlgTrig_AT-1-001-AS_3_3_sup
31. (1 point) 6.1/CCD_CCCS_Openstax_AlgTrig_AT-1-001-AS_6_1_25.p 1.pg
g Find and simplify the difference quotient for f (x) = − 2x2 + 10 .

Determine whether the table could represent a function that is Your answer must be simplified for it to be counted correct.
linear, exponential, or neither. If the function is exponential or
linear, find a function that passes through the points. If the func- f (x + h) − f (x)
tion is neither, type NONE. = help (numbers)
h
x 1 2 3 4
Answer(s) submitted:
f (x) 80 61 73 52

•? (incorrect)
• linear Correct Answers:
• exponential • -4*x-2*h
• neither

f (x) = help (formulas) 34. (1 point) 2.1/CCD_CCCS_Openstax_AlgTrig_AT-1-001-AS_2_1_5.pg


Find the x-intercept and y-intercept of y = −4x + 3.
Answer(s) submitted:
• Enter a point as (a, b), including the parentheses.

(incorrect) x-intercept: help (points)
Correct Answers: y-intercept: help (points)
• neither
• NONE Answer(s) submitted:

32. (1 point) 6.2/CCD_CCCS_Openstax_AlgTrig_AT-1-001-AS_6_2_7.pg •


The graph of f (x) = 3x is reflected across the x-axis, shifted •
downward 4 units, and then shifted left 2 units. (incorrect)
Correct Answers:
Let g(x) represent the new function. What is an equation for this
function? g(x) = help (formulas) • (0.75,0)
• (0,3)
What is the y-intercept of g(x)? Enter the answer as an ordered
pair. help (points)
35. (1 point) 6.3/RRCC_CCCS_Openstax_AlgTrig_AT-1-001-AS_6_3_39.
What is the domain of g(x)? Enter the answer as an interval. pg
help (intervals) Use the definition of logarithms to completely simplify.

What is the range of g(x)? Enter the answer as an interval. eln(1.89)


help (intervals)
help (numbers)
Answer(s) submitted:
• Answer(s) submitted:
• •

• (incorrect)
(incorrect) Correct Answers:
Correct Answers: • 1.89
• -3ˆ(x+2)-4
• (0,-13)
7
36. (1 point) 6.4/RRCC_CCCS_Openstax_AlgTrig_AT-1-001-AS_6_4_7.pg 37. (1 point) 6.6/CCD_CCCS_Openstax_AlgTrig_AT-1-001-AS_6_6_41.pg
Find the domain and range of f (x) = ln(10 − x). Enter your so- Give all solutions to the equation, or state that there are no solu-
lutions in interval notation. tions by entering “NONE”.

Domain: help (intervals) If there is more than one solution, enter the answers as a comma-
separated list.
Range: help (intervals)
ln (x − 5) − ln (x) = ln (48)
Answer(s) submitted:
• x= help (numbers)

Answer(s) submitted:
(incorrect)
Correct Answers: •
• (-infinity,10) (incorrect)
• (-infinity,infinity) Correct Answers:
• none

Generated by ©WeBWorK, http://webwork.maa.org, Mathematical Association of America

You might also like